How do I write this in summation notation?

Click For Summary
The discussion focuses on expressing a series involving sine and cosine in summation notation, specifically for an odd integer n. The series is characterized by terms of the form (\sin x)(\cos x) raised to decreasing powers, terminating when the cosine exponent reaches zero. There is confusion regarding the termination condition, as it is stated both that the series ends when the cosine exponent is 1 and when it is 0. The proposed summation notation does not align with the expected last term, which should only be sin(x). Clarification is needed on whether the series is intended solely for odd n or also for n in the form of 2^m + 1.
PhizKid
Messages
477
Reaction score
2

Homework Statement


(\sin x) (\cos x)^{n - 1} + (\sin x) (\cos x)^{\frac{n - 1}{2}} + (\sin x) (\cos x)^{\frac{\frac{n - 1}{2}}{2}} + (\sin x) (\cos x)^{\frac{\frac{\frac{n - 1}{2}}{2}}{2}} ...

n is an odd number and the series ends when (n-1)/2^k = 1, and the last term ends up being sin(x) only (no cos factor). For example, if n was 5, then 4 / 2^2 would be the last term in the series, so it looks like cos^1(x)sin(x) would be the last term, but the last term is actually sin(x) only. So if n was 5, the series would look like:

(\sin x) (\cos x)^4 + (\sin x) (\cos x)^{2} + (\sin x)


Homework Equations


N/A

The Attempt at a Solution


I got \sin(x)\sum_{k=0}^{m}\cos^{2^{m-k}}(x) for n=2^m+1 but the last term in this equation is sin(x)cos(x), not sin(x).
 
Last edited:
Physics news on Phys.org
You can't write it the way you want to because you have an inconsistency. You say that series ends when the exponent of cosine is 1, but you also say that the series ends when the exponent of cosine is 0.
 
Also, do you want this to be for just odd n as you state at the beginning? Or n = 2^m + 1 as you state later? If the former case, generic odd n will almost never result in \frac{n-1}{2^k} = 1, so what is your termination condition in that case?
 
Question: A clock's minute hand has length 4 and its hour hand has length 3. What is the distance between the tips at the moment when it is increasing most rapidly?(Putnam Exam Question) Answer: Making assumption that both the hands moves at constant angular velocities, the answer is ## \sqrt{7} .## But don't you think this assumption is somewhat doubtful and wrong?

Similar threads

  • · Replies 14 ·
Replies
14
Views
2K
  • · Replies 5 ·
Replies
5
Views
2K
  • · Replies 6 ·
Replies
6
Views
2K
  • · Replies 1 ·
Replies
1
Views
2K
  • · Replies 11 ·
Replies
11
Views
1K
  • · Replies 3 ·
Replies
3
Views
2K
  • · Replies 2 ·
Replies
2
Views
2K
  • · Replies 11 ·
Replies
11
Views
3K
  • · Replies 5 ·
Replies
5
Views
2K